Konstruieren Sie eine SO(3)SO(3)SO(3)-Rotation innerhalb zweier SU(2)SU(2)SU(2)-Grundrotationen

Die beiden kennen wir S U ( 2 ) Grundlagen haben Multiplikationszerlegungen, so dass

(1) 2 2 = 1 3.
Insbesondere hat die 3 eine Vektordarstellung von S Ö ( 3 ) . Während die 1 die triviale Darstellung von ist S U ( 2 ) .

Ich hoffe, das genaue zu sehen S Ö ( 3 ) Rotation von den beiden S U ( 2 ) Grundrotationen.

Lassen Sie uns also zuerst zwei schreiben S U ( 2 ) fundamentale Objekte im Sinne von an S Ö ( 3 ) Objekt. Insbesondere können wir die folgenden drei berücksichtigen:

| 1 , 1 = ( 1 0 ) ( 1 0 ) = | ↑↑ ,
| 1 , 0 = 1 2 ( ( 1 0 ) ( 0 1 ) + ( 0 1 ) ( 1 0 ) ) = 1 2 ( | ↑↓ + | ↓↑ ) ,
| 1 , 1 = ( 0 1 ) ( 0 1 ) = | ↓↓ .

bei dem die | Und sind in S U ( 2 ) Grundlagen. Und wir schossen | ↑↑ | | usw.

Frage: Wie rotieren wir zwischen | 1 , 1 , | 1 , 0 , | 1 , 1 , über zwei S U ( 2 ) Drehungen, die auf zwei wirken S U ( 2 ) Grundlagen? Konstruieren Sie nämlich eine S Ö ( 3 ) Rotation innerhalb der beiden S U ( 2 ) Grundrotationen? Der S U ( 2 ) hat drei Generatoren, parametrisiert durch M X , M j , M z ; wie schreiben wir das Generikum auf S Ö ( 3 ) Drehungen von zwei S U ( 2 ) Drehungen?

Betrachten wir ein Beispiel, an S U ( 2 ) Drehung U wirken auf die S U ( 2 ) grundlegend ( 1 0 ) Anlass geben

U ( 1 0 ) = ( cos ( θ 2 ) + ich M z Sünde ( θ 2 ) ( ich M X M j ) Sünde ( θ 2 ) ( ich M X + M j ) Sünde ( θ 2 ) cos ( θ 2 ) ich M z Sünde ( θ 2 ) ) ( 1 0 ) = ( cos ( θ 2 ) + ich M z Sünde ( θ 2 ) ( ich M X + M j ) Sünde ( θ 2 ) ) cos ( θ 2 ) + ich M z Sünde ( θ 2 ) ( 1 0 ) + ( ich M X + M j ) Sünde ( θ 2 ) ( 0 1 )

Mit anderen Worten, die S U ( 2 ) Drehung U (mit dem | M | 2 = 1 ) dreht S U ( 2 ) Grundlagen. Zwei S U ( 2 ) Drehungen wirken als

U U | 1 , 1 = U ( 1 0 ) U ( 1 0 ) = ( cos ( θ 2 ) + ich M z Sünde ( θ 2 ) ( ich M X + M j ) Sünde ( θ 2 ) ) ( cos ( θ 2 ) + ich M z Sünde ( θ 2 ) ( ich M X + M j ) Sünde ( θ 2 ) )

Tipp: Naiverweise bauen wir gerne

L ± = L X ± ich L j ,
so dass L ± ist ein Operator von zwei S U ( 2 ) Drehungen, die auf zwei wirken S U ( 2 ) Fundamentaldaten, so dass es dazwischen anhebt/absenkt | 1 , 1 , | 1 , 0 , | 1 , 1 .

Frage 2: Ist es plausibel, dass zwei S U ( 2 ) sind unmöglich, solche durchzuführen S Ö ( 3 ) Drehungen, aber wir brauchen zwei U ( 2 ) Drehungen?

Antworten (2)

Die folgende Lösung stammt aus der Theorie der geometrischen Quantisierung. Ich werde nicht die vollständige Theorie dahinter erklären, aber ich werde hier die Lösung geben und dann kurz diskutieren, wie man überprüft, ob dies die erforderliche Lösung ist.

Ein General S U ( 2 ) Gruppenelement in der Fundamentaldarstellung kann geschrieben werden als:

G = [ a β β ¯ a ¯ ]
mit
| a | 2 + β | 2 = 1
Der dreidimensionale Hilbert-Raum der dreidimensionalen Darstellung kann parametrisiert werden durch:
ψ ( z ) = A + B z + C z 2 ( 1 )

Wo X ist ein Unbestimmtes

Die Aktion von S U ( 2 ) auf diesem Vektorraum ist gegeben durch:

( G ψ ) ( z ) = ( β ¯ z + a ¯ ) 2 ψ ( a z + β β ¯ z + a ¯ ) ( 2 )

  1. Um zu sehen, dass dies eine Darstellung ist, kann man überprüfen, ob die Zusammensetzung der Wirkung zweier Gruppenelemente mit der Wirkung ihres Produkts übereinstimmt.
  2. Um zu sehen, dass dies ein Gläubiger ist S Ö ( 3 ) Vertretung, aber kein Gläubiger S U ( 2 ) , können wir das für das nichttriviale Element des Zentrums leicht sehen:
    G C = [ 1 0 0 1 ]
    Wir haben für jeden ψ
    ( G C ψ ) ( z ) = ψ ( z )
  3. Obwohl die "kugelförmigen" Komponenten A , B , C sind komplex. Um zu sehen, dass die Darstellung echt ist, kann man sehen, dass die "kartesischen" Komponenten ( A + C ) , B , ich 1 ( A C ) Transformation durch nur reelle Kombinationen von a Und β .
@ David Bar Moshe, danke für die Antwort, +1. Ihre Antwort ist auf einem fortgeschritteneren Niveau. :)
Dann können Sie das vielleicht auch lösen oder zumindest Ihre Meinung teilen: math.stackexchange.com/q/2745276 - danke.
@David, es sieht gut aus
U U | 1 , 1 = U ( 1 0 ) U ( 1 0 ) = ( cos ( θ 2 ) + ich M z Sünde ( θ 2 ) ( ich M X + M j ) Sünde ( θ 2 ) ) ( cos ( θ 2 ) + ich M z Sünde ( θ 2 ) ( ich M X + M j ) Sünde ( θ 2 ) )
Wenn M j = ± 1 , M X = M z = 1 Und θ = π , wir haben
U U | 1 , 1 = ( 0 1 ) ( 0 1 ) = | 1 , 1
Haben Sie eine genaue Antwort, wie Sie finden können U , so dass
U U | 1 , 1 = 1 2 ( 1 0 ) ( 0 1 ) + 1 2 ( 1 0 ) ( 0 1 ) = | 1 , 0 ?
@annie heart so eins gibt es nicht U . Bitte beachten Sie das, wenn Sie nach einigen handeln U U auf dem höchsten Gewichtsvektor: ( 1 0 ) ( 1 0 ) , dann ist das Ergebnis immer ein trennbarer Vektor der Form ( A B ) ( A B ) . Aber der Vektor, den Sie erreichen möchten, ist verschränkt, und es gibt keine Möglichkeit, den Verschränkungszustand durch eine lokale Transformation zu ändern.
Tatsächlich ist die Verschränkungsstruktur anders, wie wäre es dann, die Rotationen aus zwei U(2)-Rotationen zu konstruieren, anstatt aus zwei SU(2)?
NEIN, U ( 2 ) wird die Verschränkungsstruktur auch nicht ändern, An S U ( 3 ) Element, das auf den dreidimensionalen Vektorraum wirkt, benötigt.

Vielleicht ist folgendes hilfreich:

  1. OPs Gl. (1) ist als Beziehung zwischen komplexen Darstellungen von zu verstehen S U ( 2 ) , dh komplexe Vektorräume. Unter Hinweis darauf, dass die grundlegende S U ( 2 ) Darstellung 2 2 ¯ isomorph zur komplex konjugierten Darstellung ist, betrachten wir stattdessen den Isomorphismus

    (A) 2 2 ¯     1 3 .

  2. Die linke Seite von Gl. (A) kann als reeller Vektorraum realisiert werden u ( 2 ) von 2 × 2 Hermitesche Matrizen. Die Gruppe S U ( 2 ) wirkt auf u ( 2 ) über Konjugation. Spinor gegeben | ψ 2 , Dann

    (B) 1 S u ( 2 )     u ( 2 )     | ψ ψ |   =   1 2 μ = 0 3 X μ σ μ , ( X 0 , X 1 , X 2 , X 3 )     R 4 .
    Das Drilling 3 entspricht dem spurlosen Teil, also: S u ( 2 ) . Daher der Spinner | ψ repräsentiert den 3-Vektor R = ( X 1 , X 2 , X 3 ) . Siehe auch diesen verwandten Phys.SE-Beitrag.

Danke! +1, die gleiche Frage wie bei David, hast du eine genaue Drehung, wie man U findet, so dass UU|1,1⟩=(1/√2) (10)(01)+(1/√2) √(10)(01)=|1,0⟩ durch irgendein U?